2000 PMWC Problems/Problem T1

Revision as of 21:29, 21 April 2014 by TheMaskedMagician (talk | contribs) (Created page with "==Problem== A box contains <math>4000</math> to <math>6000</math> candies. When the candies are evenly distributed to <math>5</math>, <math>6</math>, <math>7</math>, <math>8</mat...")
(diff) ← Older revision | Latest revision (diff) | Newer revision → (diff)

Problem

A box contains $4000$ to $6000$ candies. When the candies are evenly distributed to $5$, $6$, $7$, $8$, or $9$ children, there is always one candy left. If the candies are packaged into small bags each having the same number of candies, what is the largest number of candies below $4000$ in each bag so that no candies are left?

Solution

See Also